Two PDE for one matrix-valued unknown?Two PDE for one unknown?What's a measure valued solution of a PDE?Which one of the two solutions is right?Solving $4u_tt-3u_xt-u_xx=0$Wave equation on finite domainHow to solve this PDE $partial_t u - Delta u + cu + dcdot nabla u = f$ with Dirichlet boundary conditions?Find solution of the PDE $partial_t u - partial _xxu = (x^2-2pi x)exp(-t)$ with given boundary conditionsTwo independent solutions for diffussion equation?Rewriting a linear system of PDEs as a single PDE in one dependent variable.An estimate for a 1d hyperbolic PDETwo PDE for one unknown?

Mathematica command that allows it to read my intentions

Gatling : Performance testing tool

Why do bosons tend to occupy the same state?

Extract rows of a table, that include less than x NULLs

Why no variance term in Bayesian logistic regression?

Short story with a alien planet, government officials must wear exploding medallions

What about the virus in 12 Monkeys?

Examples of smooth manifolds admitting inbetween one and a continuum of complex structures

Expand and Contract

If human space travel is limited by the G force vulnerability, is there a way to counter G forces?

How do I know where to place holes on an instrument?

What is the difference between 仮定 and 想定?

Little known, relatively unlikely, but scientifically plausible, apocalyptic (or near apocalyptic) events

Why doesn't using multiple commands with a || or && conditional work?

Avoiding direct proof while writing proof by induction

How do I gain back my faith in my PhD degree?

Why would the Red Woman birth a shadow if she worshipped the Lord of the Light?

Im going to France and my passport expires June 19th

What's the in-universe reasoning behind sorcerers needing material components?

How can saying a song's name be a copyright violation?

What does “the session was packed” mean in this context?

How can I determine if the org that I'm currently connected to is a scratch org?

How would I stat a creature to be immune to everything but the Magic Missile spell? (just for fun)

Alternative to sending password over mail?



Two PDE for one matrix-valued unknown?


Two PDE for one unknown?What's a measure valued solution of a PDE?Which one of the two solutions is right?Solving $4u_tt-3u_xt-u_xx=0$Wave equation on finite domainHow to solve this PDE $partial_t u - Delta u + cu + dcdot nabla u = f$ with Dirichlet boundary conditions?Find solution of the PDE $partial_t u - partial _xxu = (x^2-2pi x)exp(-t)$ with given boundary conditionsTwo independent solutions for diffussion equation?Rewriting a linear system of PDEs as a single PDE in one dependent variable.An estimate for a 1d hyperbolic PDETwo PDE for one unknown?













2












$begingroup$


Let $x in (0,L)$, $t in (0,T)$, and let $P = P(x,t) in mathbbR^3times 3$, $Q = Q(x,t) in mathbbR^3times 3$, $R_0 = R_0(x) in mathbbR^3times 3$ and $G= G(t) in mathbbR^3 times 3$ be continuous functions.



My question is:




Can we find a function $R = R(x,t) in mathbbR^3 times 3$ that satisfies
beginequation
begincases
partial_t R(x,t) = R(x,t) P(x,t) & textin (0,L)times(0,T)\
partial_x R(x,t) = R(x,t) Q(x,t) & textin (0,L)times(0,T)\
R(x,0) = R_0(x) & textfor x in (0,L)\
R(0,t) = G(t) & textfor t in (0,T).
endcases
endequation

with the supplementary assumptions:
beginalign*
& 1. text $R$ is unitary (i.e. $RR^intercal = R^intercal R = I$, where $I$ is the identity matrix),\
& 2. text the determinant of $R$ is equal to one,\
& 3. text $P$ and $Q$ are both skew-symmetric (i.e. $Q^intercal = -Q$ and $P^intercal = -P$).\
endalign*




(Here $partial_t$ and $partial_x$ denote the partial derivative with respect to time and space respectively.)



Following ideas from the scalar case (see Two PDE for one unknown?), I first considered, for $x in (0, L)$ fixed, the solution to the initial value problem $partial_t R = R P$:
beginalign*
R(x,t) = R_0(x) expleft( int_0^t P(x,s)ds right),
endalign*

and for $t in (0,T)$ fixed, the solution to the initial value problem $partial_x R = R Q$:
beginalign*
R(x,t) = G(t) exp left( int_0^x Q(y,t) dy right).
endalign*

I was looking for conditions on the coefficients $P$, $Q$ and on the initial and boundary data, sufficient to imply the existence of a solution $R$ (e.g. $G'(t) = G(t) Q(0,t)$, and others). However, the fact that matrices (and not scalars) are involved implies that terms do not necessarily commute (e.g. $R$ and $partial_t R$ do not necessarily commute).



Any suggestion of method, reference, explanation of why it is/is not possible to have a solution, would be welcome. Thank you.



Edit: I realize now that the formula I gave (above) for the solutions of the initial value problems are probably not correct without assuming that some quantities involving $P(x,t)$ and $Q(x,t)$ commute.










share|cite|improve this question











$endgroup$
















    2












    $begingroup$


    Let $x in (0,L)$, $t in (0,T)$, and let $P = P(x,t) in mathbbR^3times 3$, $Q = Q(x,t) in mathbbR^3times 3$, $R_0 = R_0(x) in mathbbR^3times 3$ and $G= G(t) in mathbbR^3 times 3$ be continuous functions.



    My question is:




    Can we find a function $R = R(x,t) in mathbbR^3 times 3$ that satisfies
    beginequation
    begincases
    partial_t R(x,t) = R(x,t) P(x,t) & textin (0,L)times(0,T)\
    partial_x R(x,t) = R(x,t) Q(x,t) & textin (0,L)times(0,T)\
    R(x,0) = R_0(x) & textfor x in (0,L)\
    R(0,t) = G(t) & textfor t in (0,T).
    endcases
    endequation

    with the supplementary assumptions:
    beginalign*
    & 1. text $R$ is unitary (i.e. $RR^intercal = R^intercal R = I$, where $I$ is the identity matrix),\
    & 2. text the determinant of $R$ is equal to one,\
    & 3. text $P$ and $Q$ are both skew-symmetric (i.e. $Q^intercal = -Q$ and $P^intercal = -P$).\
    endalign*




    (Here $partial_t$ and $partial_x$ denote the partial derivative with respect to time and space respectively.)



    Following ideas from the scalar case (see Two PDE for one unknown?), I first considered, for $x in (0, L)$ fixed, the solution to the initial value problem $partial_t R = R P$:
    beginalign*
    R(x,t) = R_0(x) expleft( int_0^t P(x,s)ds right),
    endalign*

    and for $t in (0,T)$ fixed, the solution to the initial value problem $partial_x R = R Q$:
    beginalign*
    R(x,t) = G(t) exp left( int_0^x Q(y,t) dy right).
    endalign*

    I was looking for conditions on the coefficients $P$, $Q$ and on the initial and boundary data, sufficient to imply the existence of a solution $R$ (e.g. $G'(t) = G(t) Q(0,t)$, and others). However, the fact that matrices (and not scalars) are involved implies that terms do not necessarily commute (e.g. $R$ and $partial_t R$ do not necessarily commute).



    Any suggestion of method, reference, explanation of why it is/is not possible to have a solution, would be welcome. Thank you.



    Edit: I realize now that the formula I gave (above) for the solutions of the initial value problems are probably not correct without assuming that some quantities involving $P(x,t)$ and $Q(x,t)$ commute.










    share|cite|improve this question











    $endgroup$














      2












      2








      2





      $begingroup$


      Let $x in (0,L)$, $t in (0,T)$, and let $P = P(x,t) in mathbbR^3times 3$, $Q = Q(x,t) in mathbbR^3times 3$, $R_0 = R_0(x) in mathbbR^3times 3$ and $G= G(t) in mathbbR^3 times 3$ be continuous functions.



      My question is:




      Can we find a function $R = R(x,t) in mathbbR^3 times 3$ that satisfies
      beginequation
      begincases
      partial_t R(x,t) = R(x,t) P(x,t) & textin (0,L)times(0,T)\
      partial_x R(x,t) = R(x,t) Q(x,t) & textin (0,L)times(0,T)\
      R(x,0) = R_0(x) & textfor x in (0,L)\
      R(0,t) = G(t) & textfor t in (0,T).
      endcases
      endequation

      with the supplementary assumptions:
      beginalign*
      & 1. text $R$ is unitary (i.e. $RR^intercal = R^intercal R = I$, where $I$ is the identity matrix),\
      & 2. text the determinant of $R$ is equal to one,\
      & 3. text $P$ and $Q$ are both skew-symmetric (i.e. $Q^intercal = -Q$ and $P^intercal = -P$).\
      endalign*




      (Here $partial_t$ and $partial_x$ denote the partial derivative with respect to time and space respectively.)



      Following ideas from the scalar case (see Two PDE for one unknown?), I first considered, for $x in (0, L)$ fixed, the solution to the initial value problem $partial_t R = R P$:
      beginalign*
      R(x,t) = R_0(x) expleft( int_0^t P(x,s)ds right),
      endalign*

      and for $t in (0,T)$ fixed, the solution to the initial value problem $partial_x R = R Q$:
      beginalign*
      R(x,t) = G(t) exp left( int_0^x Q(y,t) dy right).
      endalign*

      I was looking for conditions on the coefficients $P$, $Q$ and on the initial and boundary data, sufficient to imply the existence of a solution $R$ (e.g. $G'(t) = G(t) Q(0,t)$, and others). However, the fact that matrices (and not scalars) are involved implies that terms do not necessarily commute (e.g. $R$ and $partial_t R$ do not necessarily commute).



      Any suggestion of method, reference, explanation of why it is/is not possible to have a solution, would be welcome. Thank you.



      Edit: I realize now that the formula I gave (above) for the solutions of the initial value problems are probably not correct without assuming that some quantities involving $P(x,t)$ and $Q(x,t)$ commute.










      share|cite|improve this question











      $endgroup$




      Let $x in (0,L)$, $t in (0,T)$, and let $P = P(x,t) in mathbbR^3times 3$, $Q = Q(x,t) in mathbbR^3times 3$, $R_0 = R_0(x) in mathbbR^3times 3$ and $G= G(t) in mathbbR^3 times 3$ be continuous functions.



      My question is:




      Can we find a function $R = R(x,t) in mathbbR^3 times 3$ that satisfies
      beginequation
      begincases
      partial_t R(x,t) = R(x,t) P(x,t) & textin (0,L)times(0,T)\
      partial_x R(x,t) = R(x,t) Q(x,t) & textin (0,L)times(0,T)\
      R(x,0) = R_0(x) & textfor x in (0,L)\
      R(0,t) = G(t) & textfor t in (0,T).
      endcases
      endequation

      with the supplementary assumptions:
      beginalign*
      & 1. text $R$ is unitary (i.e. $RR^intercal = R^intercal R = I$, where $I$ is the identity matrix),\
      & 2. text the determinant of $R$ is equal to one,\
      & 3. text $P$ and $Q$ are both skew-symmetric (i.e. $Q^intercal = -Q$ and $P^intercal = -P$).\
      endalign*




      (Here $partial_t$ and $partial_x$ denote the partial derivative with respect to time and space respectively.)



      Following ideas from the scalar case (see Two PDE for one unknown?), I first considered, for $x in (0, L)$ fixed, the solution to the initial value problem $partial_t R = R P$:
      beginalign*
      R(x,t) = R_0(x) expleft( int_0^t P(x,s)ds right),
      endalign*

      and for $t in (0,T)$ fixed, the solution to the initial value problem $partial_x R = R Q$:
      beginalign*
      R(x,t) = G(t) exp left( int_0^x Q(y,t) dy right).
      endalign*

      I was looking for conditions on the coefficients $P$, $Q$ and on the initial and boundary data, sufficient to imply the existence of a solution $R$ (e.g. $G'(t) = G(t) Q(0,t)$, and others). However, the fact that matrices (and not scalars) are involved implies that terms do not necessarily commute (e.g. $R$ and $partial_t R$ do not necessarily commute).



      Any suggestion of method, reference, explanation of why it is/is not possible to have a solution, would be welcome. Thank you.



      Edit: I realize now that the formula I gave (above) for the solutions of the initial value problems are probably not correct without assuming that some quantities involving $P(x,t)$ and $Q(x,t)$ commute.







      functional-analysis pde matrix-equations matrix-calculus linear-pde






      share|cite|improve this question















      share|cite|improve this question













      share|cite|improve this question




      share|cite|improve this question








      edited Mar 21 at 15:23







      user344045

















      asked Mar 21 at 8:13









      user344045user344045

      807




      807




















          0






          active

          oldest

          votes












          Your Answer





          StackExchange.ifUsing("editor", function ()
          return StackExchange.using("mathjaxEditing", function ()
          StackExchange.MarkdownEditor.creationCallbacks.add(function (editor, postfix)
          StackExchange.mathjaxEditing.prepareWmdForMathJax(editor, postfix, [["$", "$"], ["\\(","\\)"]]);
          );
          );
          , "mathjax-editing");

          StackExchange.ready(function()
          var channelOptions =
          tags: "".split(" "),
          id: "69"
          ;
          initTagRenderer("".split(" "), "".split(" "), channelOptions);

          StackExchange.using("externalEditor", function()
          // Have to fire editor after snippets, if snippets enabled
          if (StackExchange.settings.snippets.snippetsEnabled)
          StackExchange.using("snippets", function()
          createEditor();
          );

          else
          createEditor();

          );

          function createEditor()
          StackExchange.prepareEditor(
          heartbeatType: 'answer',
          autoActivateHeartbeat: false,
          convertImagesToLinks: true,
          noModals: true,
          showLowRepImageUploadWarning: true,
          reputationToPostImages: 10,
          bindNavPrevention: true,
          postfix: "",
          imageUploader:
          brandingHtml: "Powered by u003ca class="icon-imgur-white" href="https://imgur.com/"u003eu003c/au003e",
          contentPolicyHtml: "User contributions licensed under u003ca href="https://creativecommons.org/licenses/by-sa/3.0/"u003ecc by-sa 3.0 with attribution requiredu003c/au003e u003ca href="https://stackoverflow.com/legal/content-policy"u003e(content policy)u003c/au003e",
          allowUrls: true
          ,
          noCode: true, onDemand: true,
          discardSelector: ".discard-answer"
          ,immediatelyShowMarkdownHelp:true
          );



          );













          draft saved

          draft discarded


















          StackExchange.ready(
          function ()
          StackExchange.openid.initPostLogin('.new-post-login', 'https%3a%2f%2fmath.stackexchange.com%2fquestions%2f3156508%2ftwo-pde-for-one-matrix-valued-unknown%23new-answer', 'question_page');

          );

          Post as a guest















          Required, but never shown

























          0






          active

          oldest

          votes








          0






          active

          oldest

          votes









          active

          oldest

          votes






          active

          oldest

          votes















          draft saved

          draft discarded
















































          Thanks for contributing an answer to Mathematics Stack Exchange!


          • Please be sure to answer the question. Provide details and share your research!

          But avoid


          • Asking for help, clarification, or responding to other answers.

          • Making statements based on opinion; back them up with references or personal experience.

          Use MathJax to format equations. MathJax reference.


          To learn more, see our tips on writing great answers.




          draft saved


          draft discarded














          StackExchange.ready(
          function ()
          StackExchange.openid.initPostLogin('.new-post-login', 'https%3a%2f%2fmath.stackexchange.com%2fquestions%2f3156508%2ftwo-pde-for-one-matrix-valued-unknown%23new-answer', 'question_page');

          );

          Post as a guest















          Required, but never shown





















































          Required, but never shown














          Required, but never shown












          Required, but never shown







          Required, but never shown

































          Required, but never shown














          Required, but never shown












          Required, but never shown







          Required, but never shown







          Popular posts from this blog

          Lowndes Grove History Architecture References Navigation menu32°48′6″N 79°57′58″W / 32.80167°N 79.96611°W / 32.80167; -79.9661132°48′6″N 79°57′58″W / 32.80167°N 79.96611°W / 32.80167; -79.9661178002500"National Register Information System"Historic houses of South Carolina"Lowndes Grove""+32° 48' 6.00", −79° 57' 58.00""Lowndes Grove, Charleston County (260 St. Margaret St., Charleston)""Lowndes Grove"The Charleston ExpositionIt Happened in South Carolina"Lowndes Grove (House), Saint Margaret Street & Sixth Avenue, Charleston, Charleston County, SC(Photographs)"Plantations of the Carolina Low Countrye

          random experiment with two different functions on unit interval Announcing the arrival of Valued Associate #679: Cesar Manara Planned maintenance scheduled April 23, 2019 at 00:00UTC (8:00pm US/Eastern)Random variable and probability space notionsRandom Walk with EdgesFinding functions where the increase over a random interval is Poisson distributedNumber of days until dayCan an observed event in fact be of zero probability?Unit random processmodels of coins and uniform distributionHow to get the number of successes given $n$ trials , probability $P$ and a random variable $X$Absorbing Markov chain in a computer. Is “almost every” turned into always convergence in computer executions?Stopped random walk is not uniformly integrable

          How should I support this large drywall patch? Planned maintenance scheduled April 23, 2019 at 00:00UTC (8:00pm US/Eastern) Announcing the arrival of Valued Associate #679: Cesar Manara Unicorn Meta Zoo #1: Why another podcast?How do I cover large gaps in drywall?How do I keep drywall around a patch from crumbling?Can I glue a second layer of drywall?How to patch long strip on drywall?Large drywall patch: how to avoid bulging seams?Drywall Mesh Patch vs. Bulge? To remove or not to remove?How to fix this drywall job?Prep drywall before backsplashWhat's the best way to fix this horrible drywall patch job?Drywall patching using 3M Patch Plus Primer